A state sales tax of 6% and a local sales tax of 1% are levied in Tampa, Florida. Suppose the price of a particular car in Tampa is $15,000, and an oil change at a certain auto center is $29.Which statement is true another total cost of the car and the oil change after sales tax has been calculated?Select the correct answer

A State Sales Tax Of 6% And A Local Sales Tax Of 1% Are Levied In Tampa, Florida. Suppose The Price Of

Answers

Answer 1

We have the following:

What we must do is calculate the total cost of the car by adding its original value plus the cost of taxes, 6% and 1%

We know that the initial value is $15000, if to that we add 6% of those $15000 and equal 1%, we have

[tex]15000+15000\cdot0.06+15000\cdot0.1=15000+900+150=16050[/tex]

We do the same procedure for the oil change

[tex]29+29\cdot0.06+29\cdot0.01=29+1.74+0.29=31.03[/tex]

Therefore the correct statement is the last


Related Questions

Can you just tell me the answer to this problem I need to finish it quickly I don’t need to work lol sorry it’s #6 I need help in

Answers

The combined triangles will look as shown below:

It can be observed that the lengths of the three sides fit alongside one another perfectly. This means that the lengths are equal.

Also, the middle piece of the three triangles has no space when it aligns with the other two triangles. This means that all the angles add up to 180°.It would also be a safe assumption that the angles are equal to each other, therefore the measure of each angle will be:

[tex]\Rightarrow\frac{180}{3}=60\degree[/tex]

Therefore, it can be observed that the sides and angles of the three triangles are equal.

Given Point A, what is the coordinate for A' after the following transformation has occurred?(x, y) + (x – 5, -y + 2)A (5, 7)

Answers

So we have the point A=(5,7) and the following transformation:

[tex](x,y)\rightarrow(x-5,-y+2)[/tex]

Transformations take a point as input and return another point that usually is different than the one used as input. Since our input is (5,7) then we just need to replace 5 and 7 in place of x and y on the transformation:

[tex]\begin{gathered} A^{\prime}=(x-5,-y+2)=(5-5,-7+2)=(0,-5) \\ A^{\prime}=(0,-5) \end{gathered}[/tex]

Then, the point we are looking for is A'=(0,-5).

Financial statements use the formula working C=current Assets - current Liabilities. The formula can be written in symbols as C=A-L. Solve the formula for A.

Answers

Given that C = A - L

To solve for A, add L to both sides of the equation

C + L = A - L + L

C + L = A

=>A = C + L

NO LINKS!! Please assist me with this problem. ​

Answers

Answer:

(x -h)² +(y -k)² = r²r(h, k)

Step-by-step explanation:

You are being asked for the equation of a circle, and a description of what it is.

Points equidistant

The distance equation tells you that the distance of (x, y) from (h, k) is ...

  d = √((x -h)² +(y -k)²)

If this distance is r, the radical can be removed, and we can write the equation as ...

  (x -h)² +(y -k)² = r² . . . . formula for P(x, y)

Description

This is a circle of radius r, with a center at (x, y) = (h, k).

The equation of a circle of radius r with center at (x, y) = (x -h)² +(y -k)² = r².

Given that, P(x, y) is a distance r>0 from a fixed point C(h, k).

What is a circle equation?

The equation of circle provides an algebraic way to describe a circle, given the center and the length of the radius of a circle. The equation of a circle is different from the formulas that are used to calculate the area or the circumference of a circle.

The standard equation of a circle with center at (x1,y1) and radius r is (x-x1)²+(y-y1)²=r²

Using distance formula,

The distance between (x, y) from (h, k) is

d = √((x -h)² +(y -k)²)

If this distance is r, then we get

(x -h)² +(y -k)² = r²

Hence, the equation of a circle of radius r with center at (x, y) = (x -h)² +(y -k)² = r².

To learn more about an equation of a circle visit:

https://brainly.com/question/23799314.

#SPJ1

If Rolle's Theorem can be applied, find all values of c in the open interval (a, b) such that f '(c) = 0. (Enter your answers as a comma-separated list. If Rolle's Theorem cannot be applied, enter NA.)

Answers

Since we can apply Rolle's Theorem:

[tex]\begin{gathered} f^{\prime}(x)=-\sin (x) \\ so\colon \\ f^{\prime}(x)=0 \\ -\sin (x)=0 \end{gathered}[/tex]

Take the inverse sine of both sides:

[tex]\begin{gathered} x=\sin ^{-1}(0) \\ x=\pi n \\ n\in\Z \end{gathered}[/tex]

Since it is for the interval:

[tex]\lbrack\pi,3\pi\rbrack[/tex]

The solutions are:

[tex]x=\frac{3\pi}{2},\frac{5\pi}{2}[/tex]

Answer:

[tex]\begin{gathered} c=\frac{3\pi}{2},\frac{5\pi}{2} \\ or \\ c\approx4.71,7.85 \end{gathered}[/tex]

2) A scuba diver descends to a location that is -12 1/3 m relative to sea level. He then descends another 8 1/4 M. What is the scuba divers final location relative to sea level.
NUMBERS ONLY

Answers

Answer: -20 7/12

Step-by-step explanation: Hope this helps!

One bar of candy A and two bars of candy B have 782 calories. Two bars of candy A and one bar of candy B contain 787 calories. Find the caloric content of eachcandy barCandy bar A contains calories and candy bar B contains calories

Answers

ANSWER:

Candy bar A: 264 calories

Candy bar B: 259 calories

STEP-BY-STEP EXPLANATION:

Let x be the number of calories in candy bar A and y be the number of calories in candy bar B.

We can establish the following system of equations according to the data of the statement:

[tex]\begin{gathered} x+2y=782\rightarrow x=782-2y \\ \\ 2x+y=787 \end{gathered}[/tex]

We substitute the first equation into the second and solve for y, just like this:

[tex]\begin{gathered} 2\cdot(782-2y)+y=787 \\ \\ 1564-4y+y=787 \\ \\ -3y=787-1564 \\ \\ y=\frac{-777}{-3} \\ \\ y=259 \\ \\ \text{ Now, for x:} \\ \\ x=782-2y \\ \\ x=782-2\cdot259 \\ \\ x=782-518 \\ \\ x=264 \end{gathered}[/tex]

Therefore:

Candy bar A contains 264 calories and candy bar B contains 259 calories

Let's say you have a bag with 12 cherries, 4 of the cherries are sweet and 8 are sour. If you pick a cherry atrandom, what is the probability that it will be sweet? Write your answer as a reduced fraction.Pot)

Answers

1) A rolled die has just 6 outcomes; from 1 to 6

[tex]\text{Probability = }\frac{number\text{ of required events}}{nu\text{mber of total events}}[/tex]

Number of total events for a die = 6

[tex]\begin{gathered} a)\text{ p(6) } \\ \text{for this the number of required events = 1 because there can and there is only one six showing at a time} \\ p(6)\text{ =}\frac{1}{6} \end{gathered}[/tex][tex]\begin{gathered} b)\text{ p(even)} \\ Here\text{ number of total events are 1,2,3,4,5 and 6} \\ \text{The number of even numbers = 3} \\ \\ p(\text{even) =}\frac{3}{6}=\frac{1}{2} \end{gathered}[/tex][tex]\begin{gathered} c)p(\text{greater than 1)} \\ \text{Here total number of outcomes are 1,2,3,4,5 and 6} \\ \text{numbers greater than 1 are 2,3,4,5 and 6}\ldots..\text{ Th}ere\text{ are 5 of them} \\ \text{Hence} \\ p(\text{greater than 1) =}\frac{5}{6} \end{gathered}[/tex][tex]\begin{gathered} 2)\text{ Total number of cherries = 12} \\ p(\text{sweet) =}\frac{number\text{ of sw}eet\text{ cherries}}{Total\text{number of cherries}} \\ \text{number of swe}et\text{ cherries= 4} \\ p(\text{sweet) =}\frac{4}{12}=\frac{1}{3} \end{gathered}[/tex]

In circle U m ∠TUs=107. Solve for x if m TS = (3x+39). If necessary round your answer to the nearest tenth

Answers

Answer:

x=22.7

Explanation:

In the circle:

• m∠TUS=107°

,

• The measure of arc TS = (3x+39)°

In a circle:

Therefore:

[tex]\begin{gathered} m\widehat{TS}=m\angle TUS \\ \implies3x+39=107\degree \end{gathered}[/tex]

We solve the equation for x:

[tex]\begin{gathered} \text{ Subtract 39 from both sides} \\ 3x+39-39=107-39 \\ 3x=68 \\ \text{ Divide both sides by 3} \\ \frac{3x}{3}=\frac{68}{3} \\ x\approx22.7\degree \end{gathered}[/tex]

The value of x is 22.7 (correct to the nearest tenth).

An open topped box can be created by cutting congruent squares from each of the four corners of a piece of cardboard that has dimensions of 30 cm by 40 cm and folding up the sides. Determine the dimensions of the squares that must be cut to create a box with volume of 2448cm^3.

Answers

Let's start by drawing the situation:

According to this, one of the dimensions of the box is 40-2x. The other one is 30-2x and the last one, that we could say it's the height, is x.

The volume of a box is given by the product of the three dimensions:

[tex]\begin{gathered} V=(40-2x)\cdot(30-2x)\cdot x \\ V=(1200-80x-60x+4x^2)\cdot x \\ V=1200x-140x^2+4x^3 \end{gathered}[/tex]

Use the given value of the volume to find x:

[tex]\begin{gathered} 2448=1200x-140x^2+4x^3 \\ 4x^3-140x^2+1200x-2448=0 \end{gathered}[/tex]

Factoring this expression we have that:

[tex]\begin{gathered} 4(x-3)(x^2-32x+204)=0 \\ x-3=0 \\ x=3 \end{gathered}[/tex]

One of the possible dimensions of the square is 3. Now, solve the quadratic expression (third factor) to find the other 2 options:

[tex]\begin{gathered} x^2-32x+204=0 \\ x=\frac{-(-32)\pm\sqrt[]{(-32)^2-4(1\cdot204)}}{2\cdot1} \\ x=\frac{32\pm\sqrt[]{1024-816}}{2} \\ x=\frac{32\pm\sqrt[]{208}}{2} \\ x1=\frac{32+\sqrt[]{208}}{2} \\ x2=\frac{32-\sqrt[]{208}}{2} \end{gathered}[/tex]

It means that the squares can have 3 different dimensions, which are:

[tex]3,\frac{32+\sqrt[]{208}}{2},\frac{32-\sqrt[]{208}}{2}[/tex]

Nevertheless, the second possible option is not coherent since it's value is close to 23 and the dimensions of the cardboard are 30 and 40. It means that the possible dimensions are 3 and (32-sqrt(208))/2.

10.{(0,8), (1, 2), (3, 7), (5,9), (3, 6)}

Answers

the image is downloading, please don't close the session

Solve 2sin (2x) + 2 = 0 on the interval [0, 27).π 3π 9π 11π8' 85π 7π4 4π 9π8' 85π 7π 13π 15π8' 8' 8' 8

Answers

[tex]\begin{gathered} 2\sin \mleft(2x\mright)+\sqrt{2}=0 \\ 2\sin \mleft(2x\mright)=-\sqrt{2} \\ \sin \mleft(2x\mright)=-\frac{\sqrt{2}}{2} \\ \text{General solution for }\sin \mleft(2x\mright)=-\frac{\sqrt{2}}{2} \\ 2x=\frac{5\pi}{4}+2\pi n,\: 2x=\frac{7\pi}{4}+2\pi n \\ x=\frac{5\pi}{8}+\pi n,\: x=\frac{7\pi}{8}+\pi n \end{gathered}[/tex]

7. The distance y (miles) that an athlete training for a marathon is from home after x(hours) is shown in the figure to the right.Distance from Homea. What is the y-intercept as an ordered pair?30b. Find the slope of this line. (Be aware of the scale)200Distance (miles)(1, 10)10C. What is the equation of this linear segment?0 0.5 1.0 1.5 2.0 2.5 3.0Time (hours)

Answers

[tex]\begin{gathered} a) \\ y-intercept\text{ is \lparen0,20\rparen} \\ \\ b) \\ P1(0,20) \\ P2(1,10) \\ slope=m=\frac{y2-y1}{x2-x1}=\frac{10-20}{1-0}=\frac{-10}{1}=-10 \\ The\text{ slope is -10} \\ \\ c) \\ y=mx+b \\ m=-10 \\ y=-10x+b \\ T\text{o find b using P1\lparen0,20\rparen} \\ x=0 \\ y=20 \\ 20=-10(0)+b \\ 20=0+b \\ b=20 \\ Hence,\text{ the linear equation is} \\ y=-10x+20 \end{gathered}[/tex]

The FAA now figures the average checked bag to weigh 30 pounds. This is up from a previous figureof 23 pounds. Find the amount of increase and the percent of increase, to the nearest wholepercent.

Answers

As given by the question

There are given that the average checked bag to weighs 30 pounds.

Now,

From the question

The increasing amount is:

[tex]30-23=7[/tex]

Then,

The percent of increasing is:

[tex]\begin{gathered} \frac{30-23}{23}\times100=\frac{7}{23}\times100 \\ =30.43 \end{gathered}[/tex]

Hence, the increasing amount is 7 and the percent of the increasing amount is 30%.

in a table that shows no exact solutions, how do you know if there are any solutions? How can you find an approximate solution?

Answers

If we have a quadratic equation described in a table and it does not show the exact solution (roots) of the equation, we can look if, with the values of x or the independent variable sorted, we have a change of sign.

This indicates that there is a root between those two values of x.

For example:

x = 2 --> f(x) = -3

x = 3 --> f(x) = 4

We can see that from x=2 to x=3, we have a sign change. Then we know that, because of the continuity of the quadratic function, we must have a value between x=2 and x=3 for which f(x)=0. This is an application of the Intermediate Value Theorem.

We can then approximate the value of the root x=r as the average between x=2 and x=3. This is the bisection method to find roots of functions. In this case, it would give a result r=2.5.

There are other methods (Newton-Raphson or False position, for example), but this bisection method is the simplest approximation.

Ezra has a square brick patio. He wants to reduce the width by 6 feet and increase the length by 6 feet.Let xrepresent the length of one side of the square patio Write expressions for the length and width of the new patio. Then find the area of the new patio if the original patio measures 13 feet by 13 feet.

Answers

Explanation:

Ezra has a square brick patio where x represents the length of one side of this square patio:

And hee wants to reduce the width by 6 feet, and increase the length by 6 feet.

The new width is x-6,

The new length is x+6.

[tex]\begin{gathered} Expressions\text{ for the length and the width of the new patio:} \\ l=x+6 \\ w=x-6 \end{gathered}[/tex]

Then, we need to find the area of the new patio:

[tex]A=lw[/tex]

We multiply the length by the width. The area is:

[tex]lw=(x+6)(x-6)[/tex]

And finally, if the original measure of the sides is x=13 ft, the area of the new patio is:

[tex]\begin{gathered} (x+6)(x-6) \\ \downarrow \\ (13+6)(13-6) \\ \downarrow \\ (19)(7) \\ \downarrow \\ 133 \end{gathered}[/tex]

Since it represents the area the units are square feet:

133 square feet.

These results are shown in option B.

Answer:

[tex]lw=(x+6)(x-6);133\text{ square feet}[/tex]

Which statement regarding the association shown could explain the relationship?A. class size appears to have little effect on test scores.B. schools is more affluent areas have larger class sizes, which is associated with higher test scores.C. schools in more affluent areas have smaller class sizes, which is associated with higher test scores. D. schools in less affluent areas have smaller class sizes, which is associated with lower test scores.

Answers

[tex]\begin{gathered} \text{From the scatterplot, we can s}ee, \\ \text{The size of the class is depent on the test score.} \\ \text{Higher the test score obtain in the smaller classes.} \\ Thus,\text{the answer C is correct.} \end{gathered}[/tex]

A giant pie is created in an attempt to break a world record for baking. The pie is shown below:What is the area of the slice of pie that was cut, rounded to the nearest hundredth? 78.13 ft2 82.43 ft2 86.31 ft2 91.98 ft2

Answers

step 1

Find out the area of the complete pie

[tex]A=pi*r^2[/tex]

r=30/2=15 ft ----> the radius is half the diameter

substitute

[tex]\begin{gathered} A=pi*15^2 \\ A=225pi\text{ ft}^2 \end{gathered}[/tex]

Remember that the area of a complete circle, subtends a central angle of 360 degrees

so

Applying proportion

Find out the area for a central angle of 42 degrees

[tex]\begin{gathered} \frac{225pi}{360^o}=\frac{x}{42^o} \\ \\ x=\frac{225p\imaginaryI}{360^{o}}*42^o \\ \\ x=26.25pi \\ x=26.25*3.14 \\ x=82.43\text{ ft}^2 \end{gathered}[/tex]The answer is 82.43 ft2

The last one! Synthetic division please explain how to do this!!

Answers

Answer:

Explanation:

The given expression is:

[tex](x^3+5x^2-18)\div(x-3)[/tex]

This can be solved using the synthetic division as shown below

Therefore, the quotient = x² + 8x + 24

The remainder = 54

To confirm the remainder, substitute if f(54) = 0

f(x) = x³ + 5x^2

if 2/3n = -12 what is tbe vLue of n=

Answers

If 2/3n = -12

This can be re-written as

2n/3 = -12

Multiply both sides of the equation by 3 to eliminate the fraction on the left hand side

2n = -36

Divide both sides of the equation by 2 (to eliminate the 2 and isolate the n)

n = -18

I need to know the new equation, I’ve provided a picture

Answers

Hello there. To solve this question, we'll simply have to make x => x + 5 in the function.

Given the function:

[tex]f(x)=4x^2-3^{}[/tex]

We have to determine f(x + 5)

By making x => x + 5 in this function, we get:

[tex]f(x+5)=4\cdot(x+5)^2-3[/tex]

Now remember the binomial expansion of order 2:

[tex](a+b)^2=a^2+2ab+b^2[/tex]

Therefore we have:

[tex]f(x+5)=4\cdot(x^2+2\cdot x\cdot5+5^2)-3[/tex]

Multiply the terms inside parentheses and calculate the square.

[tex]f(x+5)=4\cdot(x^2_{}+10x+25)-3[/tex]

Apply the distributive property

[tex]f(x+5)=4x^2+4\cdot10x+4\cdot25-3[/tex]

Multiply and add the numbers

[tex]\begin{gathered} f(x+5)=4x^2+40x+100-3 \\ \boxed{f(x+5)=4x^2+40x+97} \end{gathered}[/tex]

This is the answer we're looking for.

A way of showing this is the correct answer is to make x = 1 and x = 6 in the former function:

[tex]\begin{gathered} f(1)=4\cdot1^2-3=4\cdot1-3=4-3=1 \\ f(6)=4\cdot6^2-3=4\cdot36-3=144-3=141 \end{gathered}[/tex]

Then making x = 1 in the expression we found after:

[tex]f(1+5)=f(6)=4\cdot1^2+40\cdot1+97=4+40+97=141[/tex]

As expected.

which rules describe the pattern shown in the table? Select all that apply.Number of Bracelets 1. 2. 3. 4. 5Number of Beads. 16 32 48 64. 801. The number of beads is 16 times the number of bracelets.2. The number of beads is 15 more than the number of bracelets3. Each bracelet has 32 beads4. Each bracelet has 16 beads 5. The number of bracelets is equal to the number of beads.

Answers

As per given by the question.

There are given that a table of number of bracelets and numbers of beads.

Now,

According to the table,

In first option, the numbers of beads is 16 times the number of bracelets.

That means,

The number of bracelet is 1, the their 16 times greater the beads.

hence, the option first is described the pattern.

Now,

For the second option,

The number of beads is 15 more than the the number of bracelets.

So,

There are no any conclusion of option second match with the given table.

Hence, the option second is does not described the pattern.

Now,

For the option third.

Then,

According to the given table, there are different different numbers of beacelets and their different different beads. But in option third, there are given that each bracelets has 32 beads.

Hence, the option third is does not match with pattern.

Now,

For the option fourth;

The option fourth is "Each bracelets has 16 beads".

Then,

The option third is,"Each bracelets has 32 beads".

According to the given table, in all type of bracelets, atleast 16 beads are present. that means;

For 1 bracelets, there are 16 beads, for 2 bracelets, there are 32 beads(16+16), and for bracelets 3, there are 48 beads(16+16+16) so on.

Hence, the option fourth is described the pattern.

Now,

For the option fifth;

The option fourth is "the numbers of bracelets is equal to the number of beads".

Then,

According to the table, this statement is incorrect.

Hence, the option fourth also dose note described the pattern.

Then,

The option first and option fourth is described the pattern.

A cylindrical barrel has a radius of 1.2 feet and a height of 4 feet. Find the volume of the barrel.

Answers

ANSWER:

18.1 ft³

STEP-BY-STEP EXPLANATION:

Given:

Radius (r) = 1.2 ft

Height (h) = 4 ft

We can determine the volume of the cylindrical barrel using the following formula:

[tex]V=\pi\cdot r^2\cdot h[/tex]

We substitute each value and calculate the volume, like this:

[tex]\begin{gathered} V=\left(3.14\right)\left(1.2^2\right)\left(4\right) \\ \\ V=18.1\text{ ft}^3 \end{gathered}[/tex]

The volume of the barrel is 18.1 ft³

on a grid 2/3 of the squares are shaded with a color 1/4 of the squares on the grid is shaded blue what fraction of the Shaded squares are blue squares

Answers

Given:

a grid 2/3 of the squares are shaded with color.

And 1/4 of the squares on the grid is shaded blue

So, to Find the fraction of the Shaded squares are blue squares ​

Multiply both fractions

So,

[tex]\frac{2}{3}\times\frac{1}{4}=\frac{2}{12}=\frac{2}{2\cdot6}=\frac{1}{6}[/tex]

so, the answer will be 1/6 of the Shaded squares are blue squares

3
Type the correct answer in the box. Use numbers instead of words.
The number 392,000 is divided by 10.
What is the value of the digit 2 in the quotient?
Reset
Next

Answers

The value of the digit 2 in the quotient is 200

We know that,

Place value is the value of each digit in a number.

From the question, we have

392,000/10 = 39200

The value of the digit 2 in the quotient is 2 hundreds, or 200

Divide:

The simplest definition of split is to divide into two or more equally sized pieces, locations, groups, or divisions. Simply put, to divide something is to give it to a group in equal portions or to cut it into equal pieces. Consider a diagonal that creates two triangles with equal areas from a square. A division operation could result in an integer or it could not. Decimal numbers may be used to express the outcome occasionally.

To learn more about divide visit: https://brainly.com/question/15381501

#SPJ1

A training field is formed by joining a rectangle and two semicircles, as shown below. The rectangle is 91 m long and 68 m wide. What is the length of a training track running around the field? (Use the value 3.14 for I, and do not round your answer. Be sure to include the correct unit in your answer.

Answers

Answer:

Concept:

To figure out the length of the running track, we will use the following steps below

Step 1:

Calculate the length of the round the two semicircles

[tex]\begin{gathered} perimeter\text{ of semi circle=}\pi r \\ r=\frac{68m}{2}=34m \end{gathered}[/tex]

By substituting the values in the formula above, we will have

[tex]\begin{gathered} Perimeter\text{ of semicircle=}\pi r \\ Perimeter\text{ of semicircle=3.14}\times34m \\ Perimeter\text{ of semicircle=106.76m} \end{gathered}[/tex]

Step 2:

The image below will be used to calculate the length round the training track

Hence,

To calculate the length of the track we will have

[tex]\begin{gathered} Length\text{ of track=AB+arc BD+DC+arc AC} \\ AB=91m \\ arcBD=106.76m \\ arcAC=106.76m \\ DC=91m \end{gathered}[/tex]

By substituting the values, we will have

[tex]\begin{gathered} \begin{equation*} Length\text{ of track=AB+arc BD+DC+arc AC} \end{equation*} \\ Length\text{ of track=91+106.76+91m+106.76} \\ Length\text{ of track=395.52m} \\ Length\text{ of track=395.52m} \end{gathered}[/tex]

Hence,

The final answer = 395.52m

Solve for y.y+3/9=4/5

Answers

As per given by the question,

There are given that;

[tex]\frac{y-3}{9}=\frac{4}{5}[/tex]

Now,

Solve the given equation for the value of y.

So,

The given equation can be written as,

[tex]\frac{y-3}{9}-\frac{4}{5}=0[/tex]

Then,

[tex]\begin{gathered} \frac{5(y-3)-36}{45}=0 \\ 5y-15-36=0 \\ 5y-51=0 \\ 5y=51 \end{gathered}[/tex]

So,

[tex]y=\frac{51}{5}[/tex]

Hence, the value of y is;

[tex]\frac{51}{5}[/tex]

Use the Distance and Slope Formulas to complete the tables below. Round to the nearest tenth,1. Find the length of MN, given the coordinates M (4,- 4) and N (2.0).imImMN:

Answers

Given the coordinates;

[tex]\begin{gathered} M(4,-4) \\ N(2,0) \end{gathered}[/tex]

The slope m of the line MN is;

[tex]\begin{gathered} m=\frac{y_2-y_1}{x_2-x_1} \\ \text{Where x}_1=4,y_1=-4,x_2=2,y_2=0 \end{gathered}[/tex][tex]\begin{gathered} m=\frac{0-(-4)}{2-4} \\ m=\frac{4}{-2} \\ m=-2 \end{gathered}[/tex]

The slope of a line parallel to the line MN must have a slope equal to line MN, that is;

[tex]\mleft\Vert m=-2\mright?[/tex]

The slope of a line perpendicular to line MN has a slope of negative reciprocal of line MN, that is;

[tex]\begin{gathered} \perp m=-\frac{1}{-2} \\ \perp m=\frac{1}{2} \end{gathered}[/tex]

Using the distance formula to find the length of MN, the formula is given as;

[tex]D=\sqrt[]{(y_2-y_1)^2+(x_2-x_1)^2}[/tex][tex]\begin{gathered} \text{Where x}_1=4,y_1=-4,x_2=2,y_2=0 \\ |MN|=\sqrt[]{(0-(-4)^2+(2-4)^2} \\ |MN|=\sqrt[]{16+4} \\ |MN|=\sqrt[]{20} \\ |MN|=4.5 \end{gathered}[/tex]

1. Find the domain and range of f(x) = sqrt(x)2. Find the domain and range of f(x) = 3x + 2

Answers

We have the function:

[tex]f(x)=\sqrt[]{x}[/tex]

The domain is the set of values of x for which f(x) is defined. In this case, f(x) is defined only for non-negative values of x, so the domain is D:{x≥0}.

The range is the set of values that f(x) can take for the domain in which it is defined. In this case, f(x) will only take non-negative values, so the range can be defined as R: {y≥0}.

For the linear function f(x) = 3x+2, we don't have restrictions for the domain and the the range: both x and y can take any real value, so the domain and range are D: {x: all real numbers} and R: {y: all real numbers}.

Answer:

For the function f(x) = √x, the domain is D:{x≥0} and the range is R: {y≥0}.

For the function f(x) 3x+2, the domain is D: {x: all real numbers} and the range is R: {y: all real numbers}.

Write the inequality in slope - intercept form. 2x+y<13

Answers

Answer:

Step-by-step explanation:

recall the formula for slope-intercept  y=mx+b

given:   2x + y < 13

put in the equal sign but remember it's less than

2x + y = 13

y = -2x +13

now it's in slope-intercept form  :)    

Other Questions
Select the sentence from the section "U.S. targets terrorists in the Middle East" that BEST shows HOW the war on terror changed Americans' lives in the United States. Newsela The History of the War on TerrorAThe U.S. led major military attacks in Afghanistan and Iraq in the Middle East.BThey started keeping the prisoners at Guantnamo Bay, which is an American prison in Cuba.CThe National Security Agency, or NSA, began to gather more information about people.DThe country was ruled by the Taliban, an Islamic group. If tan A = 21/20 and cos B = 28/53 and angles A and B are in Quadrant I, find the valueof tan(A - B). Local and analyze Bradford use of allusions to the Bible and conferences to God's intervention in events. What purpose might this device serve in his account Our university consists of three colleges: business, engineering, and fine arts. There are 2,900 students in the business college, 1,500 students in the engineering college, and 1,000 students in the fine arts college. What percent of the total number of students are in the fine arts college. Round your answer to the nearest percent. Find the number of CDs that will produce maximum revenue. How many ounces are in 10 1/2 pounds?1 pound = 16 ounces Factor the following difference of squares. *Check for a GCF. The reaction mixture, at a certain temperature, contained concentrations of0.31 M of NH3, 0.85 M of N2 and 0.031 M of H2 when it reached equilibrium.Calculate Keq at this temperature. Transforming energy is a major trend that is reshaping agribusiness.O TrueO False christi bay water commission sends its customers an annual printout of information regarding the drinking water it supplies. this information is provided as: 8) Use the graph below to answer the question. If there are 180 students 10 pointsin the girls athletics program, then how many students participated insoccer?SPORTSSOCCER15% VOLLEYBALL20%TRACK35%BASKETBALL30%O 15O 202734 Look at the graph below and use the vertical line test to determine whether or not the graph represents a function. The determine if it is a one-to-one function.positive cube root functionThis graph Answer represent a function.This graph Answer represent a one-to-one function. The points U(1,9), V(1,5), and W(8,9) form a triangle.Plot the points then click the "Graph Triangle" button. Then find the perimeter of the triangle. Round your answer to the nearest tenth if necessary. Solve the inequality for A:-3 (9 4A) > 3 (2A 11). *Awnser this* Nikki should invest____ in the account paying 5% interest and ____ in the account paying 6.5% interest 10. A mutation in the DNA that produced the strand of messenger RNA shown below produced a newstrand of mutant messenger RNA.UGACUGCAGCUUIf the mutation was a deletion of the first base pair in the second codon, which of the following sequenceswould represent the resulting mutant messenger RNA?A. UACUGCAGCUUB. UGAUGCAGCUUC. UCUGCAGCUUD. UGAGCAGCUU Graph the function. f(x) = -3 sin x Use 3.14 for Use the sine tool to graph the function. The first point m value on the graph closest to the first point....I'll send pic of the problem After the terrorist attacks of 9/11, a long-term Democratic voter switches party to become a Republican. Since party identification is usually formed in a voter's youth and persists over time, the voter's switch is best explained bya) strong factors that convinced the voter that the Democratic Party was no longer serving his or her interests.b) the voter's party attachment did not reflect his or her views or interests.c) the voter not having developed a party attachment in youth.d) an emotional attachment to the Democratic Party. Photosynthesis is to chloroplasts as cell respiration is toPhotosynthesis is to chloroplasts as cell respiration is tocytoplasmglucosegolgi bodymitochondria 1. Observe the three graphs on frog populations in Chapter 151. What story do they tell?2. Now, look at the diagrams in Chapter 179. What do they suggest? What are the implications of this?3. Examine the timetable Christopher shares in Chapter 193. What inferences (educated guesses) canyou make about this data?4. Examine the diagram in Chapter 193 representing the relationship between space and time. Whatdoes the diagram actually say? Why does this diagram matter? What purpose does it serve?5. What is the significance of the pictures in Chapter 211? What purpose do they serve?For the curious incident of the dog in the night-time